ChaseDream
搜索
返回列表 发新帖
00:00:00

The difficulty with the proposed high-speed train line is that a used plane can be bought for one-third the price of the train line, and the plane, which is just as fast, can fly anywhere. The train would be a fixed linear system, and we live in a world that is spreading out in all directions and in which consumers choose the freewheel systems (cars, buses, aircraft), which do not have fixed routes. Thus a sufficient market for the train will not exist.

Which of the following, if true, most severely weakens the argument presented above?

正确答案: C

更多相关帖子

524

帖子

15

好友

4712

积分

ChaseDream

注册时间
2003-03-17
精华
8
解析
查看: 1506|回复: 1
打印 上一主题 下一主题

[讨论]OG-96

[复制链接]
楼主
发表于 2009-3-4 09:21:00 | 只看该作者

[讨论]OG-96

96.The difficulty with the proposed high-speed train line is that a
used plane can be bought for one-third the price of the train line, and
the plane, which is just as fast, can fly anywhere. The train would be
a fixed linear system, and we live in a world that is spreading out in
all directions and in which consumers choose the free-wheel systems
(cars, buses, aircraft), which do not have fixed routes. Thus a
sufficient market for the train will not exist.

Which of the following, if true, most severely weakens the argument presented above?
(A)
Cars, buses, and planes require the efforts of drivers and pilots to
guide them, whereas the train will be guided mechanically.
(B) Cars and buses are not nearly as fast as the high-speed train will be.
(C)
Planes are not a free-wheel system because they can fly only between
airports, which are less convenient for consumers than the high-speed
train’s stations would be.
(D) The high-speed train line cannot use currently underutilized train stations in large cities.
(E) For long trips, most people prefer to fly rather than to take ground-level transportation.

正答C,读几遍感觉都是反对前提。不是说WEAKEN只能削弱推理过程和结论么?

看了http://forum.chasedream.com/dispbbs.asp?BoardID=24&replyID=143889&id=20421&skin=0的解释,还是没看懂。
跪谢~~~
沙发
 楼主| 发表于 2009-3-4 09:37:00 | 只看该作者
顺便附上俺的“错误”推导过程:

    

前提:火车是LINEAR SYSTEM,飞机便宜且同火车一样快,“飞机是free-wheel system"

结论:火车无有效市场

正答:飞机不是FREE-WHEEL SYSTEM,没前提说的那么方便

    

因此认为:反对前提。

    

类似的例子:A骗人,B不骗人

结论:A不是好人

类似WEANKEN正答:A不是骗子

    

个人觉得这样根本无法削弱结论。因为反对前提意味着得出的结论没有站在同一个平台上,如何削弱呢?跪谢NN
您需要登录后才可以回帖 登录 | 立即注册

Mark一下! 看一下! 顶楼主! 感谢分享! 快速回复:

手机版|ChaseDream|GMT+8, 2024-9-17 05:44
京公网安备11010202008513号 京ICP证101109号 京ICP备12012021号

ChaseDream 论坛

© 2003-2023 ChaseDream.com. All Rights Reserved.

返回顶部